2016 AMC 12B Problems/Problem 7

Revision as of 11:48, 21 February 2016 by Mathmaster2012 (talk | contribs) (Created page with "==Problem== Josh writes the numbers <math>1,2,3,\dots,99,100</math>. He marks out <math>1</math>, skips the next number <math>(2)</math>, marks out <math>3</math>, and contin...")
(diff) ← Older revision | Latest revision (diff) | Newer revision → (diff)

Problem

Josh writes the numbers $1,2,3,\dots,99,100$. He marks out $1$, skips the next number $(2)$, marks out $3$, and continues skipping and marking out the next number to the end of the list. Then he goes back to the start of his list, marks out the first remaining number $(2)$, skips the next number $(4)$, marks out $6$, skips $8$, marks out $10$, and so on to the end. Josh continues in this manner until only one number remains. What is that number?

$\textbf{(A)}\ 13 \qquad \textbf{(B)}\ 32 \qquad \textbf{(C)}\ 56 \qquad \textbf{(D)}\ 64 \qquad \textbf{(E)}\ 96$

Solution

See Also

2016 AMC 12B (ProblemsAnswer KeyResources)
Preceded by
Problem 6
Followed by
Problem 8
1 2 3 4 5 6 7 8 9 10 11 12 13 14 15 16 17 18 19 20 21 22 23 24 25
All AMC 12 Problems and Solutions

The problems on this page are copyrighted by the Mathematical Association of America's American Mathematics Competitions. AMC logo.png